Prove: Derivative Proof: f(x)=0 in (0,1)

In summary, we can use the Mean Value Theorem to show that if f(0) = 0 and |f'(x)| <= |f(x)|, then f(x) = 0 for x in (0,1). By assuming f(c) > 0 for some c in (0,1), we can show that f(c) < f(c*) whenever c* < c, leading to a contradiction since f must be increasing towards f(c) but f(0) = 0. A similar proof can be made for f(c) < 0. Additionally, we can use the MVT multiple times to construct a sequence (xn) and show that it leads to a contradiction, proving that f(x
  • #1
JG89
728
1

Homework Statement



Suppose f is a differentiable function. Prove that if f(0) = 0 and |f'(x)| <= |f(x)| then f(x) = 0 for x in (0,1).

Homework Equations


The Attempt at a Solution



The actual question asks me to prove f(x) = 0 for all real x, but if I can prove f(x) = 0 for x in (0,1) then the rest follows easily.

So:

Assume there exists a c in (0,1) such that f(c) > 0. Then by the MVT there exists a c* in (0,c) such that [tex] \frac{f(c) - f(0)}{c-0} = \frac{f(c)}{c} = f'(c*) [/tex]. We know that [tex] f'(c*) \le f(c*) [/tex]. So [tex] \frac{f(c)}{c} \le f(c*) \Leftrightarrow f(c) \le cf(c*) < f(c*) [/tex] since c is in (0,1).

It follows that f(c) < f(c*) whenever c* < c. But this means that the function is decreasing. However, f(0) = 0 and from our assumption that f(c) > 0, this must mean that f must be increasing towards f(c), which is a contradiction. The proof for assuming f(c) < 0 is similar.

I think the proof is a bit shaky in the last paragraph. How is it?
 
Last edited:
Physics news on Phys.org
  • #2
I can't view the latex in my post for some reason now. It was fine when I first posted it. Now I just see a banner saying physicsforums.com wherever the latex was before.

Here is a repost of my proof:


The actual question asks me to prove f(x) = 0 for all real x, but if I can prove f(x) = 0 for x in (0,1) then the rest follows easily.

So:

Assume there exists a c in (0,1) such that f(c) > 0. Then by the MVT there exists a c* in (0,c) such that [tex] \frac{f(c) - f(0)}{c-0} = \frac{f(c)}{c} = f'(c*) [/tex]. We know that [tex] f'(c*) \le f(c*) [/tex]. So [tex] \frac{f(c)}{c} \le f(c*) \Leftrightarrow f(c) \le cf(c*) < f(c*) [/tex] since c is in (0,1).

It follows that f(c) < f(c*) whenever c* < c. But this means that the function is decreasing. However, f(0) = 0 and from our assumption that f(c) > 0, this must mean that f must be increasing towards f(c), which is a contradiction. The proof for assuming f(c) < 0 is similar.

I think the proof is a bit shaky in the last paragraph. How is it?
 
  • #3
Although you have the right idea, I don't think the proof works as stands. For one, [tex]|a| \leq |b|[/tex] does not imply [tex]a \leq b,[/tex] and I think this makes many of your inequalities very tenuous.
 
  • #4
We've assumed f(c) > 0, and c is also positive since c is in (0,1). So f(c)/c = f'(c*), making f'(c*) also positive and so [tex] |f'(c*)| \le |f(c*)| \Leftrightarrow f'(c*) \le f(c*) [/tex] so in the inequality follows.

If we assume f(c) < 0, then the inequality signs work out to find the same contradiction.
 
  • #5
ok, good point. but how do you know that f(c*) won't be negative and greater in magnitude than f'(c*). i may as well have overlooked something.
 
  • #6
f(c*) can't be negative, because f(c)/c is positive, and f(c)/c = f'(c*) <= f(c*)

Remember we've assumed that |f'(x)| <= |f(x)|, but all the values we're considering right now are positive, so we can drop the absolute value bars.
 
  • #7
Err, so you're essentially precluding the possibility of there being an x in (0,c) for which f(x) < 0? Maybe I'm missing something, but I still don't see how this follows immediately from the assumptions. Yes I'm aware that f(c)/c = f'(c) <= |f(c*)|, but how do you know that the c* in (0,c) isn't so that f(c*) < 0 (note even if f(c*) < 0, it can still be true that 0 < f'(c) =|f'(c)| <= |f(c*)|).
 
  • #8
:( Now I see your point.

I'm going to catch some sleep then work on this more tomorrow. I'm sure I'm on the right path to finishing up the proof.
 
  • #9
JG89 said:

Homework Statement



Suppose f is a differentiable function. Prove that if f(0) = 0 and |f'(x)| <= |f(x)| then f(x) = 0 for x in (0,1).

Homework Equations


The Attempt at a Solution



The actual question asks me to prove f(x) = 0 for all real x, but if I can prove f(x) = 0 for x in (0,1) then the rest follows easily.

So:

Assume there exists a c in (0,1) such that f(c) > 0. Then by the MVT there exists a c* in (0,c) such that [tex] \frac{f(c) - f(0)}{c-0} = \frac{f(c)}{c} = f'(c*) [/tex]. We know that [tex] f'(c*) \le f(c*) [/tex]. So [tex] \frac{f(c)}{c} \le f(c*) \Leftrightarrow f(c) \le cf(c*) < f(c*) [/tex] since c is in (0,1).

It follows that f(c) < f(c*) whenever c* < c. But this means that the function is decreasing. However, f(0) = 0 and from our assumption that f(c) > 0, this must mean that f must be increasing towards f(c), which is a contradiction. The proof for assuming f(c) < 0 is similar.

I think the proof is a bit shaky in the last paragraph. How is it?

Well, how about carrying on using the MVT for some more times, like this:

Proof by Contradiction
Assume that there exists some [tex]x_1 \in (0, 1) : f(x_1) \neq 0[/tex].
By the MVT, we have:
[tex]\exists x_2 \in (0, x_1) : \frac{f(x_1) - f(0)}{x_1 - 0} = f'(x_2) \Rightarrow |f(x_1)| = |f'(x_2)| x_1 \leq |f(x_2)| x_1[/tex]

Since [tex]f(x_1) \neq 0[/tex], it must follow that: [tex]f(x_2) \neq 0[/tex], we then once again, apply the MTV:

[tex]\exists x_3 \in (0, x_2) : \frac{f(x_2) - f(0)}{x_2 - 0} = f'(x_3) \Rightarrow |f(x_2)| = |f'(x_3)| x_2 \leq |f(x_3)| x_2 \Rightarrow |f(x_1)| \leq |f(x_2)| x_1 \leq |f(x_3)| x_1 x_2[/tex]

And so on, you'll be able to construct a sequence (xn), such that:
[tex]\left\{ \begin{array}{c} f(x_n) \neq 0, \forall n \\ | f(x_{1}) | \leq |f(x_{k})| \prod\limits_{i = 1}^{k - 1} x_i < |f(x_{k})| x_1 ^ {k - 1}, \forall k \geq 2 \end{array} \right.[/tex]

What can you say about the sequence (xn)? Is it convergent, or divergent?

From there, can you see what contradiction it leads to?

------------------------

Hopefully you can go from here, right? :)
 
Last edited:
  • #10
The second approach you mentioned is closer to the approach I had in mind that uses JG89's idea. In fact, I think it is possible to argue directly using this idea as well.

The problem with the first approach seems to be that we cannot assume that f ' is actually integrable.

There is even a nicer approach via proof by contradiction, I think, that makes full use of the continuity of f. If you consider the right kind of closed interval with a specific delta guaranteed by uniform continuity and use the fact that a continuous function on a closed interval attains maximum and minimum values, then you easily obtain a contradiction via the mean value theorem.
 
  • #11
snipez90 said:
The second approach you mentioned is closer to the approach I had in mind that uses JG89's idea. In fact, I think it is possible to argue directly using this idea as well.

The problem with the first approach seems to be that we cannot assume that f ' is actually integrable.

Yes, my bad. I don't really know what gets into my mind. :(

Ok, stand corrected. Thanks.. :)
 
  • #12
Thanks for the help guys.

Someone showed me a simple solution to this that uses VietDao29's method, but proves it directly.
 

1. How do you prove that f(x) = 0 in the interval (0,1)?

To prove that f(x) = 0 in the interval (0,1), we can use the definition of a derivative and the squeeze theorem. We first take the limit as x approaches 0 and 1 of f(x), and use the definition of a derivative to show that it is equal to 0. Then, we use the squeeze theorem to show that the limit of f(x) is also equal to 0 at every point in between 0 and 1, thus proving that f(x) = 0 in the interval (0,1).

2. What is the squeeze theorem and how does it help in proving the derivative of f(x) = 0 in (0,1)?

The squeeze theorem states that if two functions, g(x) and h(x), are sandwiched between another function f(x) such that g(x) ≤ f(x) ≤ h(x), and the limits of g(x) and h(x) both approach the same value, then the limit of f(x) must also approach that same value. In the proof of the derivative of f(x) = 0 in (0,1), we use the squeeze theorem to show that the limit of f(x) is equal to 0 at every point in between 0 and 1, thus proving that f(x) = 0 in the interval (0,1).

3. Can you explain the definition of a derivative and how it is used in this proof?

The definition of a derivative is the limit of the change in y (Δy) over the change in x (Δx) as Δx approaches 0. In this proof, we use this definition to show that the limit of f(x) as x approaches 0 and 1 is equal to 0. This is an important step in the proof because it shows that f(x) is approaching 0 from both sides, which is necessary for the squeeze theorem to be applicable.

4. What is the significance of proving that f(x) = 0 in the interval (0,1)?

Proving that f(x) = 0 in the interval (0,1) is significant because it means that f(x) has a horizontal tangent at every point in that interval. This tells us that the slope of the tangent line at any point in the interval is equal to 0, which is an important property of derivatives. It also allows us to make conclusions about the behavior of f(x) in that interval, such as whether it is increasing or decreasing.

5. Are there any other methods to prove that f(x) = 0 in (0,1) besides using the definition of a derivative and the squeeze theorem?

Yes, there are other methods to prove that f(x) = 0 in the interval (0,1). One method is to use the mean value theorem, which states that if a function is continuous on a closed interval [a,b] and differentiable on the open interval (a,b), then there exists a point c in (a,b) where the derivative of the function is equal to the average rate of change of the function over the interval [a,b]. In this case, we can show that the derivative of f(x) is equal to 0 at every point in the interval (0,1) using the mean value theorem, thus proving that f(x) = 0 in that interval.

Similar threads

  • Calculus and Beyond Homework Help
Replies
1
Views
279
  • Calculus and Beyond Homework Help
Replies
21
Views
839
  • Calculus and Beyond Homework Help
Replies
6
Views
548
  • Calculus and Beyond Homework Help
Replies
15
Views
1K
  • Calculus and Beyond Homework Help
Replies
26
Views
2K
  • Calculus and Beyond Homework Help
Replies
2
Views
1K
  • Calculus and Beyond Homework Help
Replies
8
Views
468
  • Calculus and Beyond Homework Help
Replies
2
Views
596
  • Calculus and Beyond Homework Help
Replies
2
Views
742
  • Calculus and Beyond Homework Help
Replies
15
Views
1K
Back
Top